LSAT and Law School Admissions Forum

Get expert LSAT preparation and law school admissions advice from PowerScore Test Preparation.

 Administrator
PowerScore Staff
  • PowerScore Staff
  • Posts: 8919
  • Joined: Feb 02, 2011
|
#85603
Complete Question Explanation

The correct answer choice is (A).

Answer choice (A): This is the correct answer choice.

Answer choice (B):

Answer choice (C):

Answer choice (D):

Answer choice (E):

This explanation is still in progress. Please post any questions below!
 lday4
  • Posts: 44
  • Joined: May 05, 2016
|
#23996
I'm having some trouble seeing what makes A a more attractive answer than B. I see how the author saying - you could argue that maybe it's not fed intervention that accelerated black economic progress, it could have been changing attitudes about employment discrimination that caused the acceleration - is the author anticipating an objection and addressing it. If that's true though, isn't that also the author introducing another factor that may have influenced black economic progress (answer B)?

Thanks!
 Robert Carroll
PowerScore Staff
  • PowerScore Staff
  • Posts: 1787
  • Joined: Dec 06, 2013
|
#24077
Hi lday,

The reason answer choice (B) is incorrect is that the author is not introducing the possible correlation of federal intervention and black economic progress. Such a possible correlation has been discussed already - the sentence before the one identified in the question itself talks about this. The author thus can't be introducing the factor at that point, because the sentence starting "True, ..." presupposes that the discussion of the federal government's role has already been initiated, but that there is some further clarification necessary of the author's viewpoint about that role.

Answer choice (A) is correct because the author, having brought up a possible relation between the federal government's intervention and black economic progress, realizes that opponent of such a theory may not agree. The author is trying to head off criticism before it undermines his/her point, so the sentence is intended to anticipate an objection. In other words, "True, what I just said might be incorrect, but allow me to explain." The partial explanation occurs in the last sentence of the passage.

Robert Carroll
 jrc3813
  • Posts: 53
  • Joined: Apr 16, 2017
|
#34662
Robert Carroll wrote:Hi lday,

The reason answer choice (B) is incorrect is that the author is not introducing the possible correlation of federal intervention and black economic progress. Such a possible correlation has been discussed already - the sentence before the one identified in the question itself talks about this. The author thus can't be introducing the factor at that point, because the sentence starting "True, ..." presupposes that the discussion of the federal government's role has already been initiated, but that there is some further clarification necessary of the author's viewpoint about that role.

Answer choice (A) is correct because the author, having brought up a possible relation between the federal government's intervention and black economic progress, realizes that opponent of such a theory may not agree. The author is trying to head off criticism before it undermines his/her point, so the sentence is intended to anticipate an objection. In other words, "True, what I just said might be incorrect, but allow me to explain." The partial explanation occurs in the last sentence of the passage.

Robert Carroll
I interpreted B to mean that it was introducing a third previously undiscussed reason for economic progress, which is changing attitudes.

The overall argument seems to flow like this:

Correlation between government action and black economic progress weakens the continuity theorists argument about educational forces.

The anticipated objection is that correlation does not equal causation. So maybe a THIRD factor caused both: changing attitudes.

He then gives some reason why that could be true, but then ultimately rejects it because the law has its greatest effect on the south where attitudes haven't changed much. Therefore it must be federal intervention that caused the progress.

It seems like the authors concession does both A and B. It anticipates an objection, but then introduces the third factor to strengthen his argument that educational forces are an inadequate explanation.

Is it because he rejects this third reason?
 Francis O'Rourke
PowerScore Staff
  • PowerScore Staff
  • Posts: 471
  • Joined: Mar 10, 2017
|
#34721
Hi JRC,

You are right that, ultimately, the author rejects the idea that changing attitudes caused economic progress. Robert Carroll is right that in this specific sentence the author is not introducing a new factor, but if that doesn't convince you, let's assume that this is the first instance of the factor that you are calling the "changing attitudes" viewpoint.

In this case, the phrases "an objection" and "another factor" denote the same idea. The real question would then be whether the author included this to point to it as a possible explanation - choice (B) - or to reject it so that the author's explanation is more convincing - Choice (A). The remainder of the paragraph should lead you towards choice (A) even in this reading.

However, you still should be able to see that the author introduced factors earlier in the paragraph and the author is now, in lines 58-60, anticipating an objection to their argument; i.e. doubting the correlation between federal intervention and economic progress.
 gweatherall
  • Posts: 39
  • Joined: Jun 29, 2017
|
#38043
For this question, I was actually confused by the contrast between A and C. I chose C, because it seemed to me that either could be correct but A was making a stronger claim so C was the safer bet. Why can we eliminate C? I know that no two answers can be right at the same time, but I can't identify an actual flaw with answer choice C.
 AthenaDalton
PowerScore Staff
  • PowerScore Staff
  • Posts: 296
  • Joined: May 02, 2017
|
#38376
Hi gweatherall,

Great question! :)

The author stops short of actually conceding the point to his opponents -- he re-asserts his belief that it's correct in the last lines of the essay.

He states, for the sake of argument, that his hypothesis (that federal intervention was the cause of the acceleration of black economic progress during this time period) might be incorrect. However, he goes on to defend his position in the final lines of the passage by saying that "Title VII had its greatest effect in the South in spite of resistance from many Southern leaders." His final conclusion is that this anecdote confirms the importance of Title VII and other federal interventions in driving black economic progress.

In other words, the author acknowledges a potential weak point in his argument without conceding the point altogether.

I hope this makes sense. Good luck studying!

Athena Dalton
 gweatherall
  • Posts: 39
  • Joined: Jun 29, 2017
|
#38566
Ahhh, I see- I didn't put enough weight on the *might* there. Thanks for clearing that up for me!

Get the most out of your LSAT Prep Plus subscription.

Analyze and track your performance with our Testing and Analytics Package.